Proof for order of cyclic groups. why does it need to continue?












0












$begingroup$


In Abstract Algebra by Dummit and Foote, page 57, I have a question regarding the proof of:



Proposition 5 Let $G$ be a group, $x in G$. Let $a in mathbb{Z} - { 0 }$. If $|x| = n < infty$, $|x^{a}| = frac{n}{(n,a)}$.



Proof: Let $y =x^{a}, (n,a) = d : and : write: n = db, a=dc$. Note since $d = (n,a)$ $b$ and $c$ are relatively prime $(b,c) = 1$. To establish our prop, we must show $|y| = b$. Note that $y^{b} = x^{ab} = x^{dcb} = (x^{n})^c = 1^c = 1$



my note: I thought that the proof would end here...since by def of order, $y^{b} =1$ implies $|y| = b$ and we let $y = x^{a}$ so $|x^{a}| = b$ and since we let $n = db$, $b = frac{n}{d}$ so we have $|x^{a}| = frac{n}{d}$, which is what we needed to prove. But the proof in the book goes on.end my note



Proof continuted: By previous prop applied to $<y>$, we have that $|y|$ divides $b$. Let $k = |y|$. Then $x^{ak} = y^{k} = 1$. So by the same previous prop applied to $<x>$, $n | ak$, or using our substitutions, $db | dck$. Thus $b | ck$. Since $(b,c) = 1$. we must have that $b |k$. Since $b, k$ are positive ints that divide each other, $b=k$.



My question: Why do we need to continue after my previous note?










share|cite|improve this question











$endgroup$








  • 1




    $begingroup$
    No, $y^b =1$ only implies $|y|$ divides $b$. For example, you know that $(-1)^4 =1$, but also note that $(-1)^2 =1$, and the order of $-1$ is $2$ not $4$ [the group could be ${1, -1}$].
    $endgroup$
    – xbh
    Nov 8 '18 at 16:17


















0












$begingroup$


In Abstract Algebra by Dummit and Foote, page 57, I have a question regarding the proof of:



Proposition 5 Let $G$ be a group, $x in G$. Let $a in mathbb{Z} - { 0 }$. If $|x| = n < infty$, $|x^{a}| = frac{n}{(n,a)}$.



Proof: Let $y =x^{a}, (n,a) = d : and : write: n = db, a=dc$. Note since $d = (n,a)$ $b$ and $c$ are relatively prime $(b,c) = 1$. To establish our prop, we must show $|y| = b$. Note that $y^{b} = x^{ab} = x^{dcb} = (x^{n})^c = 1^c = 1$



my note: I thought that the proof would end here...since by def of order, $y^{b} =1$ implies $|y| = b$ and we let $y = x^{a}$ so $|x^{a}| = b$ and since we let $n = db$, $b = frac{n}{d}$ so we have $|x^{a}| = frac{n}{d}$, which is what we needed to prove. But the proof in the book goes on.end my note



Proof continuted: By previous prop applied to $<y>$, we have that $|y|$ divides $b$. Let $k = |y|$. Then $x^{ak} = y^{k} = 1$. So by the same previous prop applied to $<x>$, $n | ak$, or using our substitutions, $db | dck$. Thus $b | ck$. Since $(b,c) = 1$. we must have that $b |k$. Since $b, k$ are positive ints that divide each other, $b=k$.



My question: Why do we need to continue after my previous note?










share|cite|improve this question











$endgroup$








  • 1




    $begingroup$
    No, $y^b =1$ only implies $|y|$ divides $b$. For example, you know that $(-1)^4 =1$, but also note that $(-1)^2 =1$, and the order of $-1$ is $2$ not $4$ [the group could be ${1, -1}$].
    $endgroup$
    – xbh
    Nov 8 '18 at 16:17
















0












0








0





$begingroup$


In Abstract Algebra by Dummit and Foote, page 57, I have a question regarding the proof of:



Proposition 5 Let $G$ be a group, $x in G$. Let $a in mathbb{Z} - { 0 }$. If $|x| = n < infty$, $|x^{a}| = frac{n}{(n,a)}$.



Proof: Let $y =x^{a}, (n,a) = d : and : write: n = db, a=dc$. Note since $d = (n,a)$ $b$ and $c$ are relatively prime $(b,c) = 1$. To establish our prop, we must show $|y| = b$. Note that $y^{b} = x^{ab} = x^{dcb} = (x^{n})^c = 1^c = 1$



my note: I thought that the proof would end here...since by def of order, $y^{b} =1$ implies $|y| = b$ and we let $y = x^{a}$ so $|x^{a}| = b$ and since we let $n = db$, $b = frac{n}{d}$ so we have $|x^{a}| = frac{n}{d}$, which is what we needed to prove. But the proof in the book goes on.end my note



Proof continuted: By previous prop applied to $<y>$, we have that $|y|$ divides $b$. Let $k = |y|$. Then $x^{ak} = y^{k} = 1$. So by the same previous prop applied to $<x>$, $n | ak$, or using our substitutions, $db | dck$. Thus $b | ck$. Since $(b,c) = 1$. we must have that $b |k$. Since $b, k$ are positive ints that divide each other, $b=k$.



My question: Why do we need to continue after my previous note?










share|cite|improve this question











$endgroup$




In Abstract Algebra by Dummit and Foote, page 57, I have a question regarding the proof of:



Proposition 5 Let $G$ be a group, $x in G$. Let $a in mathbb{Z} - { 0 }$. If $|x| = n < infty$, $|x^{a}| = frac{n}{(n,a)}$.



Proof: Let $y =x^{a}, (n,a) = d : and : write: n = db, a=dc$. Note since $d = (n,a)$ $b$ and $c$ are relatively prime $(b,c) = 1$. To establish our prop, we must show $|y| = b$. Note that $y^{b} = x^{ab} = x^{dcb} = (x^{n})^c = 1^c = 1$



my note: I thought that the proof would end here...since by def of order, $y^{b} =1$ implies $|y| = b$ and we let $y = x^{a}$ so $|x^{a}| = b$ and since we let $n = db$, $b = frac{n}{d}$ so we have $|x^{a}| = frac{n}{d}$, which is what we needed to prove. But the proof in the book goes on.end my note



Proof continuted: By previous prop applied to $<y>$, we have that $|y|$ divides $b$. Let $k = |y|$. Then $x^{ak} = y^{k} = 1$. So by the same previous prop applied to $<x>$, $n | ak$, or using our substitutions, $db | dck$. Thus $b | ck$. Since $(b,c) = 1$. we must have that $b |k$. Since $b, k$ are positive ints that divide each other, $b=k$.



My question: Why do we need to continue after my previous note?







abstract-algebra group-theory






share|cite|improve this question















share|cite|improve this question













share|cite|improve this question




share|cite|improve this question








edited Dec 1 '18 at 7:46









José Carlos Santos

153k22123226




153k22123226










asked Nov 8 '18 at 16:12









JasuusJasuus

182




182








  • 1




    $begingroup$
    No, $y^b =1$ only implies $|y|$ divides $b$. For example, you know that $(-1)^4 =1$, but also note that $(-1)^2 =1$, and the order of $-1$ is $2$ not $4$ [the group could be ${1, -1}$].
    $endgroup$
    – xbh
    Nov 8 '18 at 16:17
















  • 1




    $begingroup$
    No, $y^b =1$ only implies $|y|$ divides $b$. For example, you know that $(-1)^4 =1$, but also note that $(-1)^2 =1$, and the order of $-1$ is $2$ not $4$ [the group could be ${1, -1}$].
    $endgroup$
    – xbh
    Nov 8 '18 at 16:17










1




1




$begingroup$
No, $y^b =1$ only implies $|y|$ divides $b$. For example, you know that $(-1)^4 =1$, but also note that $(-1)^2 =1$, and the order of $-1$ is $2$ not $4$ [the group could be ${1, -1}$].
$endgroup$
– xbh
Nov 8 '18 at 16:17






$begingroup$
No, $y^b =1$ only implies $|y|$ divides $b$. For example, you know that $(-1)^4 =1$, but also note that $(-1)^2 =1$, and the order of $-1$ is $2$ not $4$ [the group could be ${1, -1}$].
$endgroup$
– xbh
Nov 8 '18 at 16:17












2 Answers
2






active

oldest

votes


















1












$begingroup$

By definition, the order of $y$ is the smallest $k$ such that $y^k=e$. So, from the fact that $y^b=e$, all you can deduce is that $operatorname{ord}yleqslant b$, not that $operatorname{ord}y=b$.






share|cite|improve this answer









$endgroup$





















    0












    $begingroup$

    You need to be careful with minimality! The order of $x$ is the minimal $n$ s.t. $x^n=1$, However, what you just found is one $n$ s.t. $x^n=1$. Hence ones till needs to work through the minimality. as an example: take $1 in mathbb{Z}/2mathbb{Z}$ (where we work additively, i.e. $x^n=n*x$)then you have $2k*1=0$, but the order is $2$ since this is the MINIMAL number such that $2*1=0$. in your proof above in this case, you might have found the "order" $16$, but you then need to reduce it to $2$






    share|cite|improve this answer









    $endgroup$













      Your Answer





      StackExchange.ifUsing("editor", function () {
      return StackExchange.using("mathjaxEditing", function () {
      StackExchange.MarkdownEditor.creationCallbacks.add(function (editor, postfix) {
      StackExchange.mathjaxEditing.prepareWmdForMathJax(editor, postfix, [["$", "$"], ["\\(","\\)"]]);
      });
      });
      }, "mathjax-editing");

      StackExchange.ready(function() {
      var channelOptions = {
      tags: "".split(" "),
      id: "69"
      };
      initTagRenderer("".split(" "), "".split(" "), channelOptions);

      StackExchange.using("externalEditor", function() {
      // Have to fire editor after snippets, if snippets enabled
      if (StackExchange.settings.snippets.snippetsEnabled) {
      StackExchange.using("snippets", function() {
      createEditor();
      });
      }
      else {
      createEditor();
      }
      });

      function createEditor() {
      StackExchange.prepareEditor({
      heartbeatType: 'answer',
      autoActivateHeartbeat: false,
      convertImagesToLinks: true,
      noModals: true,
      showLowRepImageUploadWarning: true,
      reputationToPostImages: 10,
      bindNavPrevention: true,
      postfix: "",
      imageUploader: {
      brandingHtml: "Powered by u003ca class="icon-imgur-white" href="https://imgur.com/"u003eu003c/au003e",
      contentPolicyHtml: "User contributions licensed under u003ca href="https://creativecommons.org/licenses/by-sa/3.0/"u003ecc by-sa 3.0 with attribution requiredu003c/au003e u003ca href="https://stackoverflow.com/legal/content-policy"u003e(content policy)u003c/au003e",
      allowUrls: true
      },
      noCode: true, onDemand: true,
      discardSelector: ".discard-answer"
      ,immediatelyShowMarkdownHelp:true
      });


      }
      });














      draft saved

      draft discarded


















      StackExchange.ready(
      function () {
      StackExchange.openid.initPostLogin('.new-post-login', 'https%3a%2f%2fmath.stackexchange.com%2fquestions%2f2990183%2fproof-for-order-of-cyclic-groups-why-does-it-need-to-continue%23new-answer', 'question_page');
      }
      );

      Post as a guest















      Required, but never shown

























      2 Answers
      2






      active

      oldest

      votes








      2 Answers
      2






      active

      oldest

      votes









      active

      oldest

      votes






      active

      oldest

      votes









      1












      $begingroup$

      By definition, the order of $y$ is the smallest $k$ such that $y^k=e$. So, from the fact that $y^b=e$, all you can deduce is that $operatorname{ord}yleqslant b$, not that $operatorname{ord}y=b$.






      share|cite|improve this answer









      $endgroup$


















        1












        $begingroup$

        By definition, the order of $y$ is the smallest $k$ such that $y^k=e$. So, from the fact that $y^b=e$, all you can deduce is that $operatorname{ord}yleqslant b$, not that $operatorname{ord}y=b$.






        share|cite|improve this answer









        $endgroup$
















          1












          1








          1





          $begingroup$

          By definition, the order of $y$ is the smallest $k$ such that $y^k=e$. So, from the fact that $y^b=e$, all you can deduce is that $operatorname{ord}yleqslant b$, not that $operatorname{ord}y=b$.






          share|cite|improve this answer









          $endgroup$



          By definition, the order of $y$ is the smallest $k$ such that $y^k=e$. So, from the fact that $y^b=e$, all you can deduce is that $operatorname{ord}yleqslant b$, not that $operatorname{ord}y=b$.







          share|cite|improve this answer












          share|cite|improve this answer



          share|cite|improve this answer










          answered Nov 8 '18 at 16:17









          José Carlos SantosJosé Carlos Santos

          153k22123226




          153k22123226























              0












              $begingroup$

              You need to be careful with minimality! The order of $x$ is the minimal $n$ s.t. $x^n=1$, However, what you just found is one $n$ s.t. $x^n=1$. Hence ones till needs to work through the minimality. as an example: take $1 in mathbb{Z}/2mathbb{Z}$ (where we work additively, i.e. $x^n=n*x$)then you have $2k*1=0$, but the order is $2$ since this is the MINIMAL number such that $2*1=0$. in your proof above in this case, you might have found the "order" $16$, but you then need to reduce it to $2$






              share|cite|improve this answer









              $endgroup$


















                0












                $begingroup$

                You need to be careful with minimality! The order of $x$ is the minimal $n$ s.t. $x^n=1$, However, what you just found is one $n$ s.t. $x^n=1$. Hence ones till needs to work through the minimality. as an example: take $1 in mathbb{Z}/2mathbb{Z}$ (where we work additively, i.e. $x^n=n*x$)then you have $2k*1=0$, but the order is $2$ since this is the MINIMAL number such that $2*1=0$. in your proof above in this case, you might have found the "order" $16$, but you then need to reduce it to $2$






                share|cite|improve this answer









                $endgroup$
















                  0












                  0








                  0





                  $begingroup$

                  You need to be careful with minimality! The order of $x$ is the minimal $n$ s.t. $x^n=1$, However, what you just found is one $n$ s.t. $x^n=1$. Hence ones till needs to work through the minimality. as an example: take $1 in mathbb{Z}/2mathbb{Z}$ (where we work additively, i.e. $x^n=n*x$)then you have $2k*1=0$, but the order is $2$ since this is the MINIMAL number such that $2*1=0$. in your proof above in this case, you might have found the "order" $16$, but you then need to reduce it to $2$






                  share|cite|improve this answer









                  $endgroup$



                  You need to be careful with minimality! The order of $x$ is the minimal $n$ s.t. $x^n=1$, However, what you just found is one $n$ s.t. $x^n=1$. Hence ones till needs to work through the minimality. as an example: take $1 in mathbb{Z}/2mathbb{Z}$ (where we work additively, i.e. $x^n=n*x$)then you have $2k*1=0$, but the order is $2$ since this is the MINIMAL number such that $2*1=0$. in your proof above in this case, you might have found the "order" $16$, but you then need to reduce it to $2$







                  share|cite|improve this answer












                  share|cite|improve this answer



                  share|cite|improve this answer










                  answered Nov 8 '18 at 16:18









                  EnkiduEnkidu

                  1,14819




                  1,14819






























                      draft saved

                      draft discarded




















































                      Thanks for contributing an answer to Mathematics Stack Exchange!


                      • Please be sure to answer the question. Provide details and share your research!

                      But avoid



                      • Asking for help, clarification, or responding to other answers.

                      • Making statements based on opinion; back them up with references or personal experience.


                      Use MathJax to format equations. MathJax reference.


                      To learn more, see our tips on writing great answers.




                      draft saved


                      draft discarded














                      StackExchange.ready(
                      function () {
                      StackExchange.openid.initPostLogin('.new-post-login', 'https%3a%2f%2fmath.stackexchange.com%2fquestions%2f2990183%2fproof-for-order-of-cyclic-groups-why-does-it-need-to-continue%23new-answer', 'question_page');
                      }
                      );

                      Post as a guest















                      Required, but never shown





















































                      Required, but never shown














                      Required, but never shown












                      Required, but never shown







                      Required, but never shown

































                      Required, but never shown














                      Required, but never shown












                      Required, but never shown







                      Required, but never shown







                      Popular posts from this blog

                      Quarter-circle Tiles

                      build a pushdown automaton that recognizes the reverse language of a given pushdown automaton?

                      Mont Emei